LSAT and Law School Admissions Forum

Get expert LSAT preparation and law school admissions advice from PowerScore Test Preparation.

 Administrator
PowerScore Staff
  • PowerScore Staff
  • Posts: 8916
  • Joined: Feb 02, 2011
|
#27118
Complete Question Explanation
(The complete setup for this game can be found here: lsat/viewtopic.php?t=11115)

The correct answer choice is (D)

If M is offered, then from the first rule L or S but not both must offered. This creates two distinct possibilities:
June 06_M12_game#3_L6_explanations_game#4_#16_diagram_1.png
Answer choice (A) is incorrect because if S is offered with M then L cannot be offered.

Answer choice (B) is incorrect because in one scenario P is offered, but in the other P is not offered. Thus, P does not have to be offered when L is offered.

Answer choice (C) is incorrect because if L is offered with M then S cannot be offered.

Answer choice (D) is the correct answer choice. When M and L are offered, then a total of five courses are offered. When M and S are offered, then at least three courses are offered. In each instance, at least three courses are offered, and thus this is the correct answer.

Answer choice (E) is incorrect because when M and L are offered, then a total of five courses are offered. If this answer read, “at most four additional courses are offered,” then it would be correct.
You do not have the required permissions to view the files attached to this post.

Get the most out of your LSAT Prep Plus subscription.

Analyze and track your performance with our Testing and Analytics Package.